First & Second derivative of a function

Click For Summary
The discussion revolves around finding the first and second derivatives of the function Sh(t) = 30[cos(16.04t)]. Initially, there was confusion regarding the correct application of derivative rules, particularly with trigonometric functions. The first derivative was correctly identified as 16.04 * 30 * (-sin(16.04t)), while the second derivative was found to be 16.04^2 * 30 * (-cos(16.04t)). Joe clarified that he was missing key derivative rules, such as the differentiation of constants and the application of the product rule. Ultimately, he resolved his confusion with the help of the forum members.
Agent M27
Messages
169
Reaction score
0

Homework Statement


The function Sh(t) = 30[cos(16.04*)]t models the horizantal position of a pellet with respect to time.

Find the first & second derivatives of Sh(t).



Homework Equations




The Attempt at a Solution

I attached a word document because I lack the ability to put together a correctly formatted latex doc in my post. I apologize for the inconvenience. Thank you in advance.

Joe




 

Attachments

Physics news on Phys.org
Sh(t) = 30[cos(16.04*)]t
1st derivative 30cos(16.04)
2nd 0
If the t is in the cos function, then
1st 16.04*30*(-sin16.04t)
2nd 16.04^2*30*(-cos16.04t)
 
The first term in the numerator of your limit should be 30(\cos 16.04^\circ)(t+\Delta t), so \Delta t gets multiplied by the constant.
 
I didn't download the paper. I just responded to what the derivatices would be based on what is giving. You need to do the limit definition to obtain the derivatives?
 
So for my first derivative I get to an answer of delta t/delta t, which I'm sure isn't correct. Are there some rules for differentiating when trig functions are involved that differs from a function say f(x)=x^3 ? Also I posted this question incorrectly as I was going off memory the first time. In the real problem there is no limit written next to the function, does this change things at all? I figured that equation without a limit is just the slope of a secant line. Thanks in advance.

Joe
 
Last edited:


I figured it out. I was missing some rules for derivatives such as f(x)= a constant * a variable, then f'(x) =the constant. Another one was f'(x) of a constant =0. This is of course what Dustin was trying to tell me, I just couldn't put it together from that context. Thanks for your help gentlemen.

Joe
 
Question: A clock's minute hand has length 4 and its hour hand has length 3. What is the distance between the tips at the moment when it is increasing most rapidly?(Putnam Exam Question) Answer: Making assumption that both the hands moves at constant angular velocities, the answer is ## \sqrt{7} .## But don't you think this assumption is somewhat doubtful and wrong?

Similar threads

Replies
22
Views
3K
Replies
10
Views
2K
  • · Replies 3 ·
Replies
3
Views
2K
  • · Replies 1 ·
Replies
1
Views
5K
  • · Replies 5 ·
Replies
5
Views
2K
  • · Replies 7 ·
Replies
7
Views
2K
  • · Replies 2 ·
Replies
2
Views
2K
  • · Replies 5 ·
Replies
5
Views
3K
  • · Replies 1 ·
Replies
1
Views
2K
  • · Replies 2 ·
Replies
2
Views
1K